Help with $frac12 log_2 x - frac1log_2 x = frac76$Need help with this linear equationStuck on an 'advanced logarithm problem': $2 log_2 x - log_2 (x - tfrac1 2) = log_3 3$Help with system of linear equationsI need help identifying the slope for an equation.Interval of the solutions to $log_1/2log_2(frac1+2x1+x)>0$ is?Solutions of $2^x 7^1/xle 14$Does $log_2 sqrt[4]4$ exist?Laws of logarithms: why isn't $frac14log_2(8x - 56)^16 - 12 = log_2((8x-56)^16)^frac 1 4 - 12$?How to find x if $log_2(x)cdotlog_2(x+2)=4$?Simplify $n^(log_2(n)/log_2(log_2(n)))$

Welcoming 2019 Pi day: How to draw the letter π?

Does splitting a potentially monolithic application into several smaller ones help prevent bugs?

How is the Swiss post e-voting system supposed to work, and how was it wrong?

Bash: What does "masking return values" mean?

2D counterpart of std::array in C++17

Latest web browser compatible with Windows 98

Meaning of "SEVERA INDEOVI VAS" from 3rd Century slab

How to write cleanly even if my character uses expletive language?

What does it mean to make a bootable LiveUSB?

Does the statement `int val = (++i > ++j) ? ++i : ++j;` invoke undefined behavior?

Why did it take so long to abandon sail after steamships were demonstrated?

Counting certain elements in lists

I need to drive a 7/16" nut but am unsure how to use the socket I bought for my screwdriver

How to generate globally unique ids for different tables of the same database?

An Accountant Seeks the Help of a Mathematician

Where is the 1/8 CR apprentice in Volo's Guide to Monsters?

What is this large pipe coming out of my roof?

When do we add an hyphen (-) to a complex adjective word?

Sword in the Stone story where the sword was held in place by electromagnets

Is it possible / allowed to upcast ritual spells?

Official degrees of earth’s rotation per day

The use of "touch" and "touch on" in context

Is having access to past exams cheating and, if yes, could it be proven just by a good grade?

Will a pinhole camera work with instant film?



Help with $frac12 log_2 x - frac1log_2 x = frac76$


Need help with this linear equationStuck on an 'advanced logarithm problem': $2 log_2 x - log_2 (x - tfrac1 2) = log_3 3$Help with system of linear equationsI need help identifying the slope for an equation.Interval of the solutions to $log_1/2log_2(frac1+2x1+x)>0$ is?Solutions of $2^x 7^1/xle 14$Does $log_2 sqrt[4]4$ exist?Laws of logarithms: why isn't $frac14log_2(8x - 56)^16 - 12 = log_2((8x-56)^16)^frac 1 4 - 12$?How to find x if $log_2(x)cdotlog_2(x+2)=4$?Simplify $n^(log_2(n)/log_2(log_2(n)))$













3












$begingroup$


I am supposed to get $x = 8$ and $x = x^-2/3$. What did I do wrong?



enter image description here










share|cite|improve this question











$endgroup$











  • $begingroup$
    $frac 1 log x$ is not generally the same as $log frac 1 x $
    $endgroup$
    – J. W. Tanner
    Mar 11 at 3:51










  • $begingroup$
    Did you mean $x=mathbf 2^-2/3$ ?
    $endgroup$
    – J. W. Tanner
    Mar 11 at 4:06











  • $begingroup$
    Welcome to Math SE, and thanks for providing your own thoughts on the problem. But next time use MathJax as required. It's not only much neater than handwriting, it's searchable.
    $endgroup$
    – user21820
    Mar 11 at 5:45















3












$begingroup$


I am supposed to get $x = 8$ and $x = x^-2/3$. What did I do wrong?



enter image description here










share|cite|improve this question











$endgroup$











  • $begingroup$
    $frac 1 log x$ is not generally the same as $log frac 1 x $
    $endgroup$
    – J. W. Tanner
    Mar 11 at 3:51










  • $begingroup$
    Did you mean $x=mathbf 2^-2/3$ ?
    $endgroup$
    – J. W. Tanner
    Mar 11 at 4:06











  • $begingroup$
    Welcome to Math SE, and thanks for providing your own thoughts on the problem. But next time use MathJax as required. It's not only much neater than handwriting, it's searchable.
    $endgroup$
    – user21820
    Mar 11 at 5:45













3












3








3


1



$begingroup$


I am supposed to get $x = 8$ and $x = x^-2/3$. What did I do wrong?



enter image description here










share|cite|improve this question











$endgroup$




I am supposed to get $x = 8$ and $x = x^-2/3$. What did I do wrong?



enter image description here







algebra-precalculus logarithms






share|cite|improve this question















share|cite|improve this question













share|cite|improve this question




share|cite|improve this question








edited Mar 11 at 5:56









user21820

39.4k543155




39.4k543155










asked Mar 11 at 3:47









KevinKevin

496




496











  • $begingroup$
    $frac 1 log x$ is not generally the same as $log frac 1 x $
    $endgroup$
    – J. W. Tanner
    Mar 11 at 3:51










  • $begingroup$
    Did you mean $x=mathbf 2^-2/3$ ?
    $endgroup$
    – J. W. Tanner
    Mar 11 at 4:06











  • $begingroup$
    Welcome to Math SE, and thanks for providing your own thoughts on the problem. But next time use MathJax as required. It's not only much neater than handwriting, it's searchable.
    $endgroup$
    – user21820
    Mar 11 at 5:45
















  • $begingroup$
    $frac 1 log x$ is not generally the same as $log frac 1 x $
    $endgroup$
    – J. W. Tanner
    Mar 11 at 3:51










  • $begingroup$
    Did you mean $x=mathbf 2^-2/3$ ?
    $endgroup$
    – J. W. Tanner
    Mar 11 at 4:06











  • $begingroup$
    Welcome to Math SE, and thanks for providing your own thoughts on the problem. But next time use MathJax as required. It's not only much neater than handwriting, it's searchable.
    $endgroup$
    – user21820
    Mar 11 at 5:45















$begingroup$
$frac 1 log x$ is not generally the same as $log frac 1 x $
$endgroup$
– J. W. Tanner
Mar 11 at 3:51




$begingroup$
$frac 1 log x$ is not generally the same as $log frac 1 x $
$endgroup$
– J. W. Tanner
Mar 11 at 3:51












$begingroup$
Did you mean $x=mathbf 2^-2/3$ ?
$endgroup$
– J. W. Tanner
Mar 11 at 4:06





$begingroup$
Did you mean $x=mathbf 2^-2/3$ ?
$endgroup$
– J. W. Tanner
Mar 11 at 4:06













$begingroup$
Welcome to Math SE, and thanks for providing your own thoughts on the problem. But next time use MathJax as required. It's not only much neater than handwriting, it's searchable.
$endgroup$
– user21820
Mar 11 at 5:45




$begingroup$
Welcome to Math SE, and thanks for providing your own thoughts on the problem. But next time use MathJax as required. It's not only much neater than handwriting, it's searchable.
$endgroup$
– user21820
Mar 11 at 5:45










2 Answers
2






active

oldest

votes


















3












$begingroup$

Note that



$$left( log_2(x) right)^-1 neq log_2 left(x^-1 right)$$



For example, take $x = 4$. Then



$$left( log_2(x) right)^-1 = left( log_2(4) right)^-1 = 2^-1 = frac 1 2$$



but



$$log_2 left(x^-1 right) = log_2 left( frac 1 4 right) = -2$$



This is where your error lies.






share|cite|improve this answer









$endgroup$




















    3












    $begingroup$

    To get the correct answer, let $L=log_2(x).$



    Then we have $$frac 1 2 L - frac 1 L = frac 7 6.$$



    Multiply by $6L$ to get $$3L^2-6=7L.$$



    Thus $$3L^2-7L-6=0$$



    or $$(3L+2)(L-3)=0.$$



    Can you take it from here?






    share|cite|improve this answer









    $endgroup$








    • 2




      $begingroup$
      Not gonna lie, that's a pretty slick solution (even if it's just a minor trick involving a substitution - I always love it when something like that makes the work look 10 times easier :p).
      $endgroup$
      – Eevee Trainer
      Mar 11 at 4:02










    • $begingroup$
      I know how to solve in this way, I was just confused about why I got a different answer when using another approach. But still thank you for your help ;)
      $endgroup$
      – Kevin
      Mar 11 at 4:16










    Your Answer





    StackExchange.ifUsing("editor", function ()
    return StackExchange.using("mathjaxEditing", function ()
    StackExchange.MarkdownEditor.creationCallbacks.add(function (editor, postfix)
    StackExchange.mathjaxEditing.prepareWmdForMathJax(editor, postfix, [["$", "$"], ["\\(","\\)"]]);
    );
    );
    , "mathjax-editing");

    StackExchange.ready(function()
    var channelOptions =
    tags: "".split(" "),
    id: "69"
    ;
    initTagRenderer("".split(" "), "".split(" "), channelOptions);

    StackExchange.using("externalEditor", function()
    // Have to fire editor after snippets, if snippets enabled
    if (StackExchange.settings.snippets.snippetsEnabled)
    StackExchange.using("snippets", function()
    createEditor();
    );

    else
    createEditor();

    );

    function createEditor()
    StackExchange.prepareEditor(
    heartbeatType: 'answer',
    autoActivateHeartbeat: false,
    convertImagesToLinks: true,
    noModals: true,
    showLowRepImageUploadWarning: true,
    reputationToPostImages: 10,
    bindNavPrevention: true,
    postfix: "",
    imageUploader:
    brandingHtml: "Powered by u003ca class="icon-imgur-white" href="https://imgur.com/"u003eu003c/au003e",
    contentPolicyHtml: "User contributions licensed under u003ca href="https://creativecommons.org/licenses/by-sa/3.0/"u003ecc by-sa 3.0 with attribution requiredu003c/au003e u003ca href="https://stackoverflow.com/legal/content-policy"u003e(content policy)u003c/au003e",
    allowUrls: true
    ,
    noCode: true, onDemand: true,
    discardSelector: ".discard-answer"
    ,immediatelyShowMarkdownHelp:true
    );



    );













    draft saved

    draft discarded


















    StackExchange.ready(
    function ()
    StackExchange.openid.initPostLogin('.new-post-login', 'https%3a%2f%2fmath.stackexchange.com%2fquestions%2f3143253%2fhelp-with-frac12-log-2-x-frac1-log-2-x-frac76%23new-answer', 'question_page');

    );

    Post as a guest















    Required, but never shown

























    2 Answers
    2






    active

    oldest

    votes








    2 Answers
    2






    active

    oldest

    votes









    active

    oldest

    votes






    active

    oldest

    votes









    3












    $begingroup$

    Note that



    $$left( log_2(x) right)^-1 neq log_2 left(x^-1 right)$$



    For example, take $x = 4$. Then



    $$left( log_2(x) right)^-1 = left( log_2(4) right)^-1 = 2^-1 = frac 1 2$$



    but



    $$log_2 left(x^-1 right) = log_2 left( frac 1 4 right) = -2$$



    This is where your error lies.






    share|cite|improve this answer









    $endgroup$

















      3












      $begingroup$

      Note that



      $$left( log_2(x) right)^-1 neq log_2 left(x^-1 right)$$



      For example, take $x = 4$. Then



      $$left( log_2(x) right)^-1 = left( log_2(4) right)^-1 = 2^-1 = frac 1 2$$



      but



      $$log_2 left(x^-1 right) = log_2 left( frac 1 4 right) = -2$$



      This is where your error lies.






      share|cite|improve this answer









      $endgroup$















        3












        3








        3





        $begingroup$

        Note that



        $$left( log_2(x) right)^-1 neq log_2 left(x^-1 right)$$



        For example, take $x = 4$. Then



        $$left( log_2(x) right)^-1 = left( log_2(4) right)^-1 = 2^-1 = frac 1 2$$



        but



        $$log_2 left(x^-1 right) = log_2 left( frac 1 4 right) = -2$$



        This is where your error lies.






        share|cite|improve this answer









        $endgroup$



        Note that



        $$left( log_2(x) right)^-1 neq log_2 left(x^-1 right)$$



        For example, take $x = 4$. Then



        $$left( log_2(x) right)^-1 = left( log_2(4) right)^-1 = 2^-1 = frac 1 2$$



        but



        $$log_2 left(x^-1 right) = log_2 left( frac 1 4 right) = -2$$



        This is where your error lies.







        share|cite|improve this answer












        share|cite|improve this answer



        share|cite|improve this answer










        answered Mar 11 at 3:50









        Eevee TrainerEevee Trainer

        7,87121339




        7,87121339





















            3












            $begingroup$

            To get the correct answer, let $L=log_2(x).$



            Then we have $$frac 1 2 L - frac 1 L = frac 7 6.$$



            Multiply by $6L$ to get $$3L^2-6=7L.$$



            Thus $$3L^2-7L-6=0$$



            or $$(3L+2)(L-3)=0.$$



            Can you take it from here?






            share|cite|improve this answer









            $endgroup$








            • 2




              $begingroup$
              Not gonna lie, that's a pretty slick solution (even if it's just a minor trick involving a substitution - I always love it when something like that makes the work look 10 times easier :p).
              $endgroup$
              – Eevee Trainer
              Mar 11 at 4:02










            • $begingroup$
              I know how to solve in this way, I was just confused about why I got a different answer when using another approach. But still thank you for your help ;)
              $endgroup$
              – Kevin
              Mar 11 at 4:16















            3












            $begingroup$

            To get the correct answer, let $L=log_2(x).$



            Then we have $$frac 1 2 L - frac 1 L = frac 7 6.$$



            Multiply by $6L$ to get $$3L^2-6=7L.$$



            Thus $$3L^2-7L-6=0$$



            or $$(3L+2)(L-3)=0.$$



            Can you take it from here?






            share|cite|improve this answer









            $endgroup$








            • 2




              $begingroup$
              Not gonna lie, that's a pretty slick solution (even if it's just a minor trick involving a substitution - I always love it when something like that makes the work look 10 times easier :p).
              $endgroup$
              – Eevee Trainer
              Mar 11 at 4:02










            • $begingroup$
              I know how to solve in this way, I was just confused about why I got a different answer when using another approach. But still thank you for your help ;)
              $endgroup$
              – Kevin
              Mar 11 at 4:16













            3












            3








            3





            $begingroup$

            To get the correct answer, let $L=log_2(x).$



            Then we have $$frac 1 2 L - frac 1 L = frac 7 6.$$



            Multiply by $6L$ to get $$3L^2-6=7L.$$



            Thus $$3L^2-7L-6=0$$



            or $$(3L+2)(L-3)=0.$$



            Can you take it from here?






            share|cite|improve this answer









            $endgroup$



            To get the correct answer, let $L=log_2(x).$



            Then we have $$frac 1 2 L - frac 1 L = frac 7 6.$$



            Multiply by $6L$ to get $$3L^2-6=7L.$$



            Thus $$3L^2-7L-6=0$$



            or $$(3L+2)(L-3)=0.$$



            Can you take it from here?







            share|cite|improve this answer












            share|cite|improve this answer



            share|cite|improve this answer










            answered Mar 11 at 3:56









            J. W. TannerJ. W. Tanner

            3,2401320




            3,2401320







            • 2




              $begingroup$
              Not gonna lie, that's a pretty slick solution (even if it's just a minor trick involving a substitution - I always love it when something like that makes the work look 10 times easier :p).
              $endgroup$
              – Eevee Trainer
              Mar 11 at 4:02










            • $begingroup$
              I know how to solve in this way, I was just confused about why I got a different answer when using another approach. But still thank you for your help ;)
              $endgroup$
              – Kevin
              Mar 11 at 4:16












            • 2




              $begingroup$
              Not gonna lie, that's a pretty slick solution (even if it's just a minor trick involving a substitution - I always love it when something like that makes the work look 10 times easier :p).
              $endgroup$
              – Eevee Trainer
              Mar 11 at 4:02










            • $begingroup$
              I know how to solve in this way, I was just confused about why I got a different answer when using another approach. But still thank you for your help ;)
              $endgroup$
              – Kevin
              Mar 11 at 4:16







            2




            2




            $begingroup$
            Not gonna lie, that's a pretty slick solution (even if it's just a minor trick involving a substitution - I always love it when something like that makes the work look 10 times easier :p).
            $endgroup$
            – Eevee Trainer
            Mar 11 at 4:02




            $begingroup$
            Not gonna lie, that's a pretty slick solution (even if it's just a minor trick involving a substitution - I always love it when something like that makes the work look 10 times easier :p).
            $endgroup$
            – Eevee Trainer
            Mar 11 at 4:02












            $begingroup$
            I know how to solve in this way, I was just confused about why I got a different answer when using another approach. But still thank you for your help ;)
            $endgroup$
            – Kevin
            Mar 11 at 4:16




            $begingroup$
            I know how to solve in this way, I was just confused about why I got a different answer when using another approach. But still thank you for your help ;)
            $endgroup$
            – Kevin
            Mar 11 at 4:16

















            draft saved

            draft discarded
















































            Thanks for contributing an answer to Mathematics Stack Exchange!


            • Please be sure to answer the question. Provide details and share your research!

            But avoid


            • Asking for help, clarification, or responding to other answers.

            • Making statements based on opinion; back them up with references or personal experience.

            Use MathJax to format equations. MathJax reference.


            To learn more, see our tips on writing great answers.




            draft saved


            draft discarded














            StackExchange.ready(
            function ()
            StackExchange.openid.initPostLogin('.new-post-login', 'https%3a%2f%2fmath.stackexchange.com%2fquestions%2f3143253%2fhelp-with-frac12-log-2-x-frac1-log-2-x-frac76%23new-answer', 'question_page');

            );

            Post as a guest















            Required, but never shown





















































            Required, but never shown














            Required, but never shown












            Required, but never shown







            Required, but never shown

































            Required, but never shown














            Required, but never shown












            Required, but never shown







            Required, but never shown







            Popular posts from this blog

            Lowndes Grove History Architecture References Navigation menu32°48′6″N 79°57′58″W / 32.80167°N 79.96611°W / 32.80167; -79.9661132°48′6″N 79°57′58″W / 32.80167°N 79.96611°W / 32.80167; -79.9661178002500"National Register Information System"Historic houses of South Carolina"Lowndes Grove""+32° 48' 6.00", −79° 57' 58.00""Lowndes Grove, Charleston County (260 St. Margaret St., Charleston)""Lowndes Grove"The Charleston ExpositionIt Happened in South Carolina"Lowndes Grove (House), Saint Margaret Street & Sixth Avenue, Charleston, Charleston County, SC(Photographs)"Plantations of the Carolina Low Countrye

            random experiment with two different functions on unit interval Announcing the arrival of Valued Associate #679: Cesar Manara Planned maintenance scheduled April 23, 2019 at 00:00UTC (8:00pm US/Eastern)Random variable and probability space notionsRandom Walk with EdgesFinding functions where the increase over a random interval is Poisson distributedNumber of days until dayCan an observed event in fact be of zero probability?Unit random processmodels of coins and uniform distributionHow to get the number of successes given $n$ trials , probability $P$ and a random variable $X$Absorbing Markov chain in a computer. Is “almost every” turned into always convergence in computer executions?Stopped random walk is not uniformly integrable

            How should I support this large drywall patch? Planned maintenance scheduled April 23, 2019 at 00:00UTC (8:00pm US/Eastern) Announcing the arrival of Valued Associate #679: Cesar Manara Unicorn Meta Zoo #1: Why another podcast?How do I cover large gaps in drywall?How do I keep drywall around a patch from crumbling?Can I glue a second layer of drywall?How to patch long strip on drywall?Large drywall patch: how to avoid bulging seams?Drywall Mesh Patch vs. Bulge? To remove or not to remove?How to fix this drywall job?Prep drywall before backsplashWhat's the best way to fix this horrible drywall patch job?Drywall patching using 3M Patch Plus Primer